Calcular la transformada de Fourier de tiempo discreto inverso

2

Calcule la transformada de Fourier de tiempo discreto inverso de lo siguiente donde \ $ | a | < 1 \ $:

$$   X (e ^ {j \ omega}) = \ frac {1-a ^ 2} {(1-ae ^ {- j \ omega}) (1-ae ^ {j \ omega})} $$

Al insertar esto directamente en la ecuación de IDTFT, obtengo:

\ begin {align *}   x [n] & = \ frac {1} {2 \ pi} \ int _ {- \ pi} ^ \ pi X (e ^ {j \ omega}) e ^ {j \ omega n} d \ omega \\   x [n] & = \ frac {1} {2 \ pi} \ int _ {- \ pi} ^ \ pi          \ frac {(1-a ^ 2) e ^ {j \ omega n}} {(1-ae ^ {- j \ omega}) (1-ae ^ {j \ omega})} d \ omega \\ \ end {align *}

Estoy teniendo problemas para empezar. No estoy seguro de qué intentar. Ninguna de las leyes de propiedad estándar de la Transformada de Fourier parece aplicarse directamente a esto.

(Este es el problema 2.57 del libro de texto de Oppenheim sobre el procesamiento de señales de tiempo discreto)

    
pregunta clay

2 respuestas

3

Necesitamos jugar un poco con el diseño de la expresión. Tenemos:

$$ F = \ frac {(1 + a) (1-a)} {(1-ae ^ {\ jw}) (1-ae ^ {\ -jw})} = \ frac {(1 + a)} {(1-ae ^ {\ -jw})} \ frac {(1-a)} {(1-ae ^ {\ jw})} $$

Podemos reescribirlo como:

$$ = \ left (\ frac {1} {(1-ae ^ {\ -jw})} + \ frac {a} {(1-ae ^ {\ -jw})} \ right) \ left (\ frac {1} {(1-ae ^ {\ jw})} - \ frac {a} {(1-ae ^ {\ jw})} \ right) \\ $$

Factorizamos un \ $ - ae ^ {\ jw} \ $ de los términos más a la derecha y hacemos la transformación inversa:

$$ = \ left (\ frac {1} {(1-ae ^ {\ -jw})} + a \ frac {1} {(1-ae ^ {\ -jw})} \ right) \ left (- \ frac {1} {a} \ frac {e ^ {\ -jw}} {(1- \ frac {1} {a} e ^ {\ -jw})} + \ frac {e ^ {\ -jw}} {(1- \ frac {1} {a} e ^ {\ -jw})} \ right) \\ \ implica \ left (a ^ nu [n] + a (a ^ nu [n]) \ right) * \ left (- \ tfrac {1} {a} ({\ tfrac {1} {a}} ^ nu [n-1]) + ({\ tfrac {1} {a}} ^ nu [n-1]) \ right) $$

Finalmente, limpiando:

$$ = \ left (a ^ nu [n] (1 + a) \ right) * \ left (\ tfrac {1} {a} ^ {n-1} u [n-1] (1- \ tfrac {1} {a}) \ derecha) $$

Lo siento si es demasiado desordenado. Fourier suele ser mucha escritura. ¡Dime dónde crees que pueda haber cometido un error o no está claro! Si alguien encuentra un error, hágamelo saber.

    
respondido por el Cehhiro
1

¿Esto está fuera?

\ begin {align *}   X (e ^ {j \ omega}) & = X_1 (e ^ {j \ omega}) \ cdot X_2 (e ^ {j \ omega}) \\   X_1 (e ^ {j \ omega}) & = \ frac {1-a ^ 2} {1-ae ^ {- j \ omega}} \\   X_2 (e ^ {j \ omega}) & = \ frac {1} {1-ae ^ {j \ omega}} \\   x_1 [n] & = (1-a ^ 2) a ^ nu [n] \\   x_2 [n] & = a ^ {- n} u [-n] \\   x [n] & = x_1 [n] * x_2 [n] \\   x [n] & = \ sum \ limits_ {k = - \ infty} ^ \ infty x_1 [k] x_2 [n-k] \\   x [n] & = \ sum \ limits_ {k = 0} ^ \ infty (1-a ^ 2) a ^ k a ^ {k-n} u [k-n] \\   x [n] & = (1-a ^ 2) a ^ {- n} \ sum \ limits_ {k = 0} ^ \ infty (a ^ 2) ^ k u [k-n] \\   x [n] & = \ begin {cases}       (1-a ^ 2) a ^ {- n} \ sum \ limits_ {k = n} ^ \ infty (a ^ 2) ^ k & n \ ge 0 \\       (1-a ^ 2) a ^ {- n} \ sum \ limits_ {k = 0} ^ \ infty (a ^ 2) ^ k & n < 0 \\     \ end {cases} \\   x [n] & = \ begin {cases}       (1-a ^ 2) a ^ {- n} \ frac {a ^ {2n}} {1-a ^ 2} & n \ ge 0 \\       (1-a ^ 2) a ^ {- n} \ frac {1} {1-a ^ 2} & n < 0 \\     \ end {cases} \\   x [n] & = \ begin {cases}       a ^ {n} & n \ ge 0 \\       a ^ {- n} & n < 0 \\     \ end {cases} \\   x [n] & = a ^ {| n |} \\ \ end {align *}

    
respondido por el clay

Lea otras preguntas en las etiquetas